jrany12
Thanks Received: 2
Forum Guests
 
Posts: 23
Joined: October 27th, 2010
 
 
 

Q6 - Economist: As should be obvious

by jrany12 Sun Nov 28, 2010 7:01 pm

Hi,
Could you please explain why (A) is a better answer than (D)? Is (D) wrong because of the word "most"? If so, how?
Also, when the word "most" comes up in the LSAT, does it mean at least more than half (half plus one), or at least half? Thanks!
 
giladedelman
Thanks Received: 833
LSAT Geek
 
Posts: 619
Joined: April 04th, 2010
 
This post thanked 2 times.
 
 

Re: Q6 - Economist: As should be obvious

by giladedelman Tue Nov 30, 2010 12:09 am

Thanks for posting!

We've got a little intermediate conclusion on our hands here:

Premise: raising the minimum wage significantly would make it more expensive for businesses to pay workers for those jobs, THEREFORE...

Intermediate conclusion: businesses could not afford to employ as many workers, THEREFORE...

Main conclusion: raising minimum wage significantly would cause unemployment to go up.

(A) is correct because it weakens the logic between the premise and the intermediate conclusion. If businesses pass the cost of increased wages on to consumers without hurting their profits, then they should actually be able to continue to employ the same amount of workers.

(D) is incorrect because it's really irrelevant what portion of workers currently earn minimum wage. The question is, will raising the minimum wage cause unemployment to increase due to businesses having to employ fewer workers? All we care about is what happens to those workers, not the extent of the workforce they represent.

(For the record, "most" means "more than half.")

(B) is out of scope. We don't know if the first part is the case here, and the second part doesn't affect whether unemployment will increase.

(C) is tempting until we remember that the argument is not about whether this is acceptable, just about whether it will happen.

(E) is wildly incorrect. So what if unemployment has been decreasing -- we want to know what will happen!

Does that clear this one up for you?
 
jrany12
Thanks Received: 2
Forum Guests
 
Posts: 23
Joined: October 27th, 2010
 
This post thanked 1 time.
 
 

Re: PT57, S3, Q6 - Economist: As should be obvious, raising the

by jrany12 Tue Nov 30, 2010 9:00 am

For (D), I just thought if most workers already earn more than the current min wage, then raising the min wage wouldn't really make a difference to those businesses since they're already paying higher to their workers anyway. So they could afford to continue employing those workers.
But I guess since it says "most", there still could be other businesses that can't afford to, so there would still be an increase in unemployment potentially. And "significantly" isn't precisely defined, so it could mean raising min wage by a whole lot to the point where no businesses could afford to. Is this correct?

Also, does the word "typically" in A have any effect? I feel like I didn't choose it partly bc of that word, bc it's not always that businesses pass the costs to consumers, just typically. What are your thoughts?

Thanks for all your explanations!
 
giladedelman
Thanks Received: 833
LSAT Geek
 
Posts: 619
Joined: April 04th, 2010
 
This post thanked 2 times.
 
 

Re: PT57, S3, Q6 - Economist: As should be obvious, raising the

by giladedelman Fri Dec 03, 2010 12:24 am

Yes, you're on the right track here. Those modifying words are important!

For (D), if it said that every worker currently makes more than minimum wage, it would be a winner; the intermediate conclusion that businesses would have to shed jobs would no longer follow. But because it's just most workers, this doesn't really affect the logic. The businesses still have to pay for those workers who do make minimum wage.

As for (A), you're right that "always" would make it a better answer than "typically." But we're just looking for the answer that most weakens the argument, and the fact that businesses typically pass the costs on to consumers does weaken the argument somewhat -- more than we can say about any of the other answers.
User avatar
 
LSAT-Chang
Thanks Received: 38
Atticus Finch
Atticus Finch
 
Posts: 479
Joined: June 03rd, 2011
 
This post thanked 1 time.
 
trophy
Most Thankful
trophy
First Responder
 

Re: Q6 - Economist: As should be obvious, raising the

by LSAT-Chang Thu Sep 22, 2011 3:36 pm

Hey Gilad,
I have a different opinion on this one.
I don't think (D) would be wrong because of the word "MOST" because in the LSAT, most can imply "all" -- so even if we had (D) every worker is earning more than the current minimum wage -- it won't affect our argument because the stimulus says "raising the minimum wage significantly..." and we don't know how much more than the current minimum wage these "every" worker is earning. So these "every" worker could be earning just 5 cents more than the minimum wage (since that is still "more" than the minimum wage), so raising the minimum wage "significantly" would still make these businesses not be able to employ as many workers for such jobs. So (D) would still make the conclusion hold and not "weaken" it even if it had said "EVERY" worker. I would love to hear feedback if anyone disagrees.. :)
 
bsd987
Thanks Received: 0
Forum Guests
 
Posts: 4
Joined: October 21st, 2011
 
 
 

Re: Q6 - Economist: As should be obvious, raising the

by bsd987 Fri Oct 21, 2011 10:33 am

I first came across this question 9 months ago while I was studying for the LSAT, and I guess now that I'm tutoring the LSAT (privately for now at least), I need some closure.

Having read this question probably 300 times, I've gotten myself convinced that every answer choice is equally wrong, as in no choice is anything more than 0 percent correct.

As you guys have already discussed, the word "typically" is very weak and "always" would have made it stronger. But it goes beyond that in my opinion. Typically implies that it is not always; in fact, typically implies that there are examples to the contrary. Just one example to the contrary negates the claim. If one company goes against the typical occurrence and lays off just one employee, all other factors equal, the unemployment rate has increased, even if by a ridiculously small decimal.

The argument of the economist is that a significant increase in the minimum wage raises unemployment rate. There is no discussion of by how much it raises the rate. An increase for 10.000000% to 10.000001% still validates the economist's argument. All it has to do is increase slightly, and his argument is so true.

(A) does not do anything to make that statement false. Nothing at all. Therefore, the choice is entirely wrong, as wrong as the other four choices.

Maybe I've worked myself into overthinking this question, and certainly had I taken this LSAT four years ago, I would have fought to have the question thrown out (or maybe not, it wasn't an inherently difficult question).

I guess what I'm asking is for someone to convince me that my analysis is wrong. Maybe I'm just too stubborn...
 
raziel
Thanks Received: 5
Forum Guests
 
Posts: 26
Joined: January 15th, 2012
 
 
 

Re: Q6 - Economist: As should be obvious, raising the

by raziel Sun Jan 15, 2012 9:29 pm

bsd987 Wrote:I first came across this question 9 months ago while I was studying for the LSAT, and I guess now that I'm tutoring the LSAT (privately for now at least), I need some closure.

Having read this question probably 300 times, I've gotten myself convinced that every answer choice is equally wrong, as in no choice is anything more than 0 percent correct.

As you guys have already discussed, the word "typically" is very weak and "always" would have made it stronger. But it goes beyond that in my opinion. Typically implies that it is not always; in fact, typically implies that there are examples to the contrary. Just one example to the contrary negates the claim. If one company goes against the typical occurrence and lays off just one employee, all other factors equal, the unemployment rate has increased, even if by a ridiculously small decimal.

The argument of the economist is that a significant increase in the minimum wage raises unemployment rate. There is no discussion of by how much it raises the rate. An increase for 10.000000% to 10.000001% still validates the economist's argument. All it has to do is increase slightly, and his argument is so true.

(A) does not do anything to make that statement false. Nothing at all. Therefore, the choice is entirely wrong, as wrong as the other four choices.

Maybe I've worked myself into overthinking this question, and certainly had I taken this LSAT four years ago, I would have fought to have the question thrown out (or maybe not, it wasn't an inherently difficult question).

I guess what I'm asking is for someone to convince me that my analysis is wrong. Maybe I'm just too stubborn...


Don't know if this is right but here it goes:

The first sentence is the premise. The second sentence is an intermediate conclusion. The last sentence is the conclusion.

The necessary assumption that we have to make to go from the first sentence to the intermediate conclusion is that only at the current minimum wage could businesses afford to continue to afford as many workers for minimum wage jobs.

We attack it by showing another way in which businesses can continue to employ as many workers, and therefore, raising the minimum wage will not cause an increase in unemployment.

(A) Does this by showing that at least in some cases businesses can pass the cost of increased wages to consumers, and hence, if businesses can do this, they will be able to at least afford more employees than if they could not do this (thus weakened). We don't have to prove the argument is false. With (A) we have show that at least some businesses can continue to pay for as many workers as they had.
 
tplan21
Thanks Received: 5
Forum Guests
 
Posts: 13
Joined: January 20th, 2012
 
 
trophy
Most Thankful
 

Re: Q6 - Economist: As should be obvious, raising the

by tplan21 Wed Jan 25, 2012 11:48 am

I disagree with most here on why D is wrong. I might add i chose D first butnow, understand why its wrong.

The key words being "minimum wage jobs" in the stimulus.

We are only concerned with jobs that pay min wages.

D) so what if most are paid more than min wage, we only care about those that are not. Doesn't weaken the conclusion.

Initially, I picked D BC I missed the scope of only minimium wage jobs and thought jobs in general.


B) with this option the business doesn't actually suffer from the increase since its passed on to the consumer.

Initially, I didn't like this option BC i thought well then the consumer won't want to buy it since its more expensive and so it will still be affected since businesses wont recieve as much revenue as before and may have to eliminate the source of the increase, employees. But I now realize the key words "without adversly affecting profits"

Hope this helps others.
 
giladedelman
Thanks Received: 833
LSAT Geek
 
Posts: 619
Joined: April 04th, 2010
 
This post thanked 1 time.
 
 

Re: Q6 - Economist: As should be obvious, raising the

by giladedelman Fri Jan 27, 2012 11:40 am

Great posts here.

First, let me address bsd987, because you've worked yourself into quite a lather on this one! You make one crucial mistake: typically does not imply that there are examples to the contrary. It is accurate to say that the typical human breathes oxygen; this doesn't imply that there are some anomalies (got the spelling right there on my third try, by the way) who breathe methane or something.

Raziel's post is beautiful: we attack the argument by showing a way in which businesses actually could raise the minimum wage without employing fewer workers. And Raziel makes a super important point: we don't have to make the argument false, we just have to weaken it, that is, make it less likely to be true. Well, if restaurants can "typically" pass these costs onto consumers, then maybe they won't have to fire anyone! And hey, maybe not all restaurants can do this, but maybe the ones that can't have some other trick that will allow them to retain their employees. (A) makes rising unemployment less likely, and that's good enough to weaken the argument.

Changsoyeon, I think you've pointed out another nice reason to get rid of (D), in addition to the beef that jrany12 and I have with it. tplan21, I think you're actually agreeing with us!
 
jamiejames
Thanks Received: 3
Atticus Finch
Atticus Finch
 
Posts: 116
Joined: September 17th, 2011
 
 
 

Re: Q6 - Economist: As should be obvious

by jamiejames Sun Jun 03, 2012 5:13 pm

Here's why D is wrong.

Let's say that minimum wage is now set at $7.25.
Now, most workers are paid above this, let's say $8. Well, that's great, and good for them. Now, let's say the minimum wage is increased to $8.25 ( which it could very well be as the stim says "raising minimum wage significantly which could mean to $8 or to $20, who knows) now, all these works who were once working at above minimum wage are now working at it (possibly). Now, how does this happening weaken the argument that raising min wage will = increased unemployment? To my mind, it doesn't.

However, A means that any increase the company would have to pay to keepn all it's workers can be passed onto consumers, who may all have to pay 30 cents more for a product, which means profits won't be hurt (unless everyone stopped buying because of the increase but this is too big a jump,) and hence no one will loose a job, and employment won't increase.
 
sch6les
Thanks Received: 5
Forum Guests
 
Posts: 13
Joined: July 24th, 2012
 
 
 

Re: Q6 - Economist: As should be obvious

by sch6les Thu Apr 04, 2013 11:59 pm

Uh, both (A) and (D) weaken the argument. Why are people trying to show (D) doesn't weaken the argument? It absolutely does. The problem is that (A) is a better weaken than (D). There are less holes in (A). That is why (A) is correct.

The question stem states which MOST weakens.
User avatar
 
tommywallach
Thanks Received: 468
Atticus Finch
Atticus Finch
 
Posts: 1041
Joined: August 11th, 2009
 
 
 

Re: Q6 - Economist: As should be obvious

by tommywallach Sat Apr 06, 2013 12:42 pm

Hey sch6les,

Thinking you didn't read the explanations enough, because (D) definitely doesn't weaken. Believe it or not, in spite of the whole "which most weakens" thing, there's pretty much always one right answer and four wrong ones.

In this case, the conclusion of the argument is:

Raising the minimum wage significantly will cause an increase in unemployment.

(D) Most workers are earning more than minimum wage.

So there are two reasons this doesn't weaken. First of all, "most" only means 51% or more. So there could still be 40% of the workforce making minimum wage. That's a lot of people to make more money when you raise the minimum wage. Remember, even if it were a 90%/10% split, that would still have a big impact on the bottom line and thus increase unemployment. Remember, the conclusion only says there will be some increase in unemployment, not a significant one.

To repeat, the conclusion says "raising the minimum wage significantly will cause an increase in unemployment." It does not say "raising the minimum wage will significantly increase unemployment." This brings us to the second problem with (D). We know that we're raising the minimum wage by a lot. (D) says people are earning more than the minimum wage. But that could be making 10 cents more, or a dollar more. So a significant increase could still affect them.

Between these two issues, (D) cannot be said to weaken on any but the most hypothetical level. Hope that makes sense!

-t
Tommy Wallach
Manhattan LSAT Instructor
twallach@manhattanprep.com
Image
User avatar
 
ttunden
Thanks Received: 0
Atticus Finch
Atticus Finch
 
Posts: 146
Joined: August 09th, 2012
 
 
 

Re: Q6 - Economist: As should be obvious

by ttunden Fri Aug 15, 2014 5:10 pm

Here is another explanation. I got this question wrong during the PT(only question I missed from this particular LR section)

Basically the economist is arguing that businesses that currently employ minimum wage employees cannot afford to employ them much longer if minimum wage increases. Keyword being currently employ, meaning the economist is assuming or narrowing this argument only to CURRENT minimum wage employees. So, lets say we have a 100 employees earning minimum wage of $7.50, raising that to $10 will CAUSE an increase in unemployment.

So, there are several things we can do to weaken, we can attack the causation by showing cause + no effect, or no cause +effect, or an alternate cause.

A is the former with cause +no effect. They will pass the cost of increased minimum wage to customers without affecting profit.

I picked D 1st time around and I see why it's wrong. The Economist is narrowing his argument only to current minimum wage employees. Who cares if mots workers are earning more than current minimum wage. The economist is only talking about current minimum wage employees at businesses. So D won't weaken the economist argument since its out of scope. I missed this concept during the test and was only focusing on minimum wage and unemployment, I didn't realize the author narrowed the argument to only current minimum wage employees.
 
ganbayou
Thanks Received: 0
Atticus Finch
Atticus Finch
 
Posts: 213
Joined: June 13th, 2015
 
 
 

Re: Q6 - Economist: As should be obvious

by ganbayou Mon Aug 10, 2015 7:03 am

So when you say "Most workers" are out of scope, is it because in the stimulus the economist says "workers for such jobs"?
I also chose D and I think in old PT I saw "most~~"actually works to weak argument, but I do not remember which PT...
Do we have to be careful with "most" always? I read somewhere "some" is weak to weak or streghten arguments, but "most" actually works because it is more than half.
I read all explanations above and understand why A weakens and why D might not, but still wonder when "most" statement can weaken arguments bc I remember I saw somewhere.
By the way, "Pass on to consumers" mean they increase the prices of the products right?

Thank you
User avatar
 
maryadkins
Thanks Received: 641
Atticus Finch
Atticus Finch
 
Posts: 1261
Joined: March 23rd, 2011
 
 
 

Re: Q6 - Economist: As should be obvious

by maryadkins Mon Aug 10, 2015 7:44 am

"Most" is more specific than "some" because "most" means 51% or more, while "some" could mean 2 or could mean everyone/everything—you have no clue. It is only in this way that "most" is going to be a "better weakener" potentially— because it actually means something specific, it is more likely to be a good specific attack. It isn't, on its own, a good weakener by nature of being the word "most." You need to analyze it in context.

ganbayou Wrote:By the way, "Pass on to consumers" mean they increase the prices of the products right?


Yes.
 
ganbayou
Thanks Received: 0
Atticus Finch
Atticus Finch
 
Posts: 213
Joined: June 13th, 2015
 
 
 

Re: Q6 - Economist: As should be obvious

by ganbayou Mon Aug 10, 2015 1:56 pm

Thanks for the reply,
So...when you say "Most workers" are out of scope, is it because in the stimulus the economist says "workers for such jobs"? Sorry but just want to make sure :oops:
User avatar
 
tommywallach
Thanks Received: 468
Atticus Finch
Atticus Finch
 
Posts: 1041
Joined: August 11th, 2009
 
This post thanked 1 time.
 
 

Re: Q6 - Economist: As should be obvious

by tommywallach Fri Aug 14, 2015 3:53 pm

Yes.
Tommy Wallach
Manhattan LSAT Instructor
twallach@manhattanprep.com
Image
 
805218400
Thanks Received: 0
Vinny Gambini
Vinny Gambini
 
Posts: 3
Joined: March 09th, 2017
 
 
 

Re: Q6 - Economist: As should be obvious

by 805218400 Thu May 18, 2017 4:15 am

I agree with LSAT-Chang, D is wrong for "more than the current minimum wages", but not for "Most", 'cause if their earning is more than current minimum wage, like 15 (current is 10), but after raising the minimum wage, the new minimum wage is 20, in this condition, businesses still need to pay for these workers and it may happen what is mentioned in the premise. maybe if D says "more than new minimum wage", it will be much more plausible
 
YudeS218
Thanks Received: 0
Vinny Gambini
Vinny Gambini
 
Posts: 21
Joined: September 01st, 2017
 
 
 

Re: Q6 - Economist: As should be obvious

by YudeS218 Wed Jan 16, 2019 6:50 am

I think I know why D is wrong and fails to weaken the argument, however, I insist that A is wrong as well.

How we define "typically"? Is that mean 51% or more? or 90% or more? Definitely not 100%.

If it is not 100%, then there still would be an increase in unemployment, therefore the argument still holds.